Difference between revisions of "2021 AMC 10B Problems/Problem 1"

m (Solution)
(Removed all unnecessary contents. I merged all contents on the corresponding AMC 12 page.)
(Tag: Replaced)
 
(19 intermediate revisions by 13 users not shown)
Line 1: Line 1:
How many integer values of <math>x</math> satisfy <math>|x|<3\pi</math>?
+
#REDIRECT [[2021 AMC 12B Problems/Problem 1|Solution]]
 
 
<math>\textbf{(A)} ~9 \qquad\textbf{(B)} ~10 \qquad\textbf{(C)} ~18 \qquad\textbf{(D)} ~19 \qquad\textbf{(E)} ~20</math>
 
 
 
==Solution==
 
 
 
Since <math>3\pi</math> is about <math>9.42</math>, we multiply 9 by 2 and add 1 to get <math> \boxed{\textbf{(D)}\ ~19} </math>
 
~smarty101
 

Latest revision as of 05:16, 4 March 2021